Is my approach correct to this equation?












1












$begingroup$


The problem is the following:




Does $a in mathbb{R}$ exist such that $[a + sqrt{2n + 1}] = [a + sqrt{2n + 2}]$ for all $n in mathbb{N}$? ($[x]$ denotes the whole part of $x$).




Note: I will also use ${x}$ to denote the fractional part of $x$. Also that $[x] + {x} = x$.



My approach is the following:



There is no such that $a$.



Proof: For the sake of simplicity, let's assume $a = k + alpha$, $k in mathbb{Z}$ and $0 le alpha lt 1$. Because of $[x + k] = [x] + k$ for any $k in mathbb{Z}$, and substituting $a = k + alpha$, our equation $[k + alpha + sqrt{2n + 1}] = [k + alpha + sqrt{2n + 2}]$ becomes $[alpha + sqrt{2n + 1}] = [alpha + sqrt{2n + 2}]$. Now our task is to find $alpha in [0, 1)$ that satisfies the equation for any $n in mathbb{N}$.



First, let's find a formula for $alpha$ for given $n$. There are two cases:



1. $[sqrt{2n + 1}] = [sqrt{2n + 2}] = m$, $m in mathbb{N}$. This happens when $2n + 2$ isn't a perfect square $iff$ $2n + 2 ne p^2 iff n ne frac {p^2} {2} - 1$, $p in mathbb{N}$.



$[alpha + sqrt{2n + 1}] = [alpha + sqrt{2n + 2}] iff [alpha + [sqrt{2n + 1}] + {sqrt{2n + 1}}] = [alpha + [sqrt{2n + 2}] + {sqrt{2n + 2}}] iff [alpha + {sqrt{2n + 1}} + m] = [alpha + {sqrt{2n + 2}} + m] iff [alpha + {sqrt{2n + 1}}] = [alpha + {sqrt{2n + 2}}]$.

From this we get that $alpha in [0, 1 - {sqrt{2n + 2}}) tag {1'}$
Edit #1 (no longer actual beacuse of Edit #2): took out the maximum function from $(1')$ for obvious reason: the fractional part function is monotonically growing on its period, so if $[x] = [y]$, then ${x} lt {y}$ only if $x lt y$.
Edit #2: $(1')$ is only a partial solution for the 1. case, it is indeed a solution if both sides of the latest equation is equal to $0$. But they can both equal to $1$, so we have to consider that $alpha in [1 - {sqrt{2n + 1}}, 1)$. 1. case overall:
$alpha in [0, 1 - {sqrt{2n + 2}}) cup [1 - {sqrt{2n + 1}}, 1) tag 1$



2. $[sqrt{2n + 1}] ne [sqrt{2n + 2}] implies [sqrt{2n + 1}] = m, [sqrt{2n + 2}] = m + 1$, $m in mathbb{N}$. This happens when $2n + 2$ is a perfect square $iff$ $2n + 2 = p^2 iff n = frac {p^2} {2} - 1$, $p in mathbb{N}$. Now $[sqrt{2n + 2}] = sqrt{2n + 2} implies {sqrt{2n + 2}} = 0$.



$[alpha + sqrt{2n + 1}] = [alpha + sqrt{2n + 2}] iff [alpha + [sqrt{2n + 1}] + {sqrt{2n + 1}}] = [alpha + [sqrt{2n + 2}] + {sqrt{2n + 2}}] iff [alpha + {sqrt{2n + 1}}] = [alpha + {sqrt{2n + 2}}] + 1 iff [alpha + {sqrt{2n + 1}}] = [alpha] + 1 iff [alpha + {sqrt{2n + 1}}] = 1$

From this we get that $alpha = 1 - {sqrt{2n + 1}} tag 2$



Now we prove the following: (this is where it gets really ambiguous)

There does exists $n_1, n_2 in mathbb{N}$, $n_1 ne frac {{p_1}^2} {2} - 1$, $n_2 = frac {{p_2}^2} {2} - 1$, $p_1$, $p_2 in mathbb{N} iff [sqrt{2n_1 + 1}] = [sqrt{2n_1 + 2}]$, $[sqrt{2n_2 + 1}] ne [sqrt{2n_2 + 2}]$ such that $ 1 - {sqrt{2n_1 + 2}} le 1 - {sqrt{2n_2 + 1}} lt 1 - {sqrt{2n_1 + 1}} tag 3 iff$ there is no $alpha$ satisfying both $(1)$ and $(2)$.
$implies {sqrt{2n_1 + 1}} lt {sqrt{2n_2 + 1}} le {sqrt{2n_1 + 2}} tag 4$
(this is where it gets really-really-really ambiguous)




If we substitute $n_1 = 3$ and $n_2 = 1$ in $(4)$ we get the following:
${sqrt{7}} lt {sqrt{3}} le {sqrt{8}} implies approx 0.64 lt 0.73 le 0.82$ wich is indeed true. So we found two exceptions for $n$ and we should be done with the proof.




This part is strongly wrong! I made a mistake. $(4)$ is wrong. In fact, I can't find $n_1$ and $n_2$ satisfying $(3)$. Now I believe that I am wrong and there does exist a desired $a$.



Note: I made a lot of edits. My proof changed since I posted the question. Please take a look at the edits.










share|cite|improve this question











$endgroup$












  • $begingroup$
    @TheSimpliFire appears the quoted part in yellow should read "every" $n$ rather than "any" $n,$ as here the "any" is ambiguous. Now that I think of it, "all" is pretty good, then it reads "for all"
    $endgroup$
    – Will Jagy
    Jan 4 at 20:17












  • $begingroup$
    @TheSimpliFire I am afraid that one of us didn't understand something properly.
    $endgroup$
    – Krisztián Kiss
    Jan 4 at 20:18










  • $begingroup$
    @KrisztiánKiss I recommend you change the quoted phrase from "for any" to "for all"
    $endgroup$
    – Will Jagy
    Jan 4 at 20:21










  • $begingroup$
    @WillJagy thank you.
    $endgroup$
    – Krisztián Kiss
    Jan 4 at 20:23
















1












$begingroup$


The problem is the following:




Does $a in mathbb{R}$ exist such that $[a + sqrt{2n + 1}] = [a + sqrt{2n + 2}]$ for all $n in mathbb{N}$? ($[x]$ denotes the whole part of $x$).




Note: I will also use ${x}$ to denote the fractional part of $x$. Also that $[x] + {x} = x$.



My approach is the following:



There is no such that $a$.



Proof: For the sake of simplicity, let's assume $a = k + alpha$, $k in mathbb{Z}$ and $0 le alpha lt 1$. Because of $[x + k] = [x] + k$ for any $k in mathbb{Z}$, and substituting $a = k + alpha$, our equation $[k + alpha + sqrt{2n + 1}] = [k + alpha + sqrt{2n + 2}]$ becomes $[alpha + sqrt{2n + 1}] = [alpha + sqrt{2n + 2}]$. Now our task is to find $alpha in [0, 1)$ that satisfies the equation for any $n in mathbb{N}$.



First, let's find a formula for $alpha$ for given $n$. There are two cases:



1. $[sqrt{2n + 1}] = [sqrt{2n + 2}] = m$, $m in mathbb{N}$. This happens when $2n + 2$ isn't a perfect square $iff$ $2n + 2 ne p^2 iff n ne frac {p^2} {2} - 1$, $p in mathbb{N}$.



$[alpha + sqrt{2n + 1}] = [alpha + sqrt{2n + 2}] iff [alpha + [sqrt{2n + 1}] + {sqrt{2n + 1}}] = [alpha + [sqrt{2n + 2}] + {sqrt{2n + 2}}] iff [alpha + {sqrt{2n + 1}} + m] = [alpha + {sqrt{2n + 2}} + m] iff [alpha + {sqrt{2n + 1}}] = [alpha + {sqrt{2n + 2}}]$.

From this we get that $alpha in [0, 1 - {sqrt{2n + 2}}) tag {1'}$
Edit #1 (no longer actual beacuse of Edit #2): took out the maximum function from $(1')$ for obvious reason: the fractional part function is monotonically growing on its period, so if $[x] = [y]$, then ${x} lt {y}$ only if $x lt y$.
Edit #2: $(1')$ is only a partial solution for the 1. case, it is indeed a solution if both sides of the latest equation is equal to $0$. But they can both equal to $1$, so we have to consider that $alpha in [1 - {sqrt{2n + 1}}, 1)$. 1. case overall:
$alpha in [0, 1 - {sqrt{2n + 2}}) cup [1 - {sqrt{2n + 1}}, 1) tag 1$



2. $[sqrt{2n + 1}] ne [sqrt{2n + 2}] implies [sqrt{2n + 1}] = m, [sqrt{2n + 2}] = m + 1$, $m in mathbb{N}$. This happens when $2n + 2$ is a perfect square $iff$ $2n + 2 = p^2 iff n = frac {p^2} {2} - 1$, $p in mathbb{N}$. Now $[sqrt{2n + 2}] = sqrt{2n + 2} implies {sqrt{2n + 2}} = 0$.



$[alpha + sqrt{2n + 1}] = [alpha + sqrt{2n + 2}] iff [alpha + [sqrt{2n + 1}] + {sqrt{2n + 1}}] = [alpha + [sqrt{2n + 2}] + {sqrt{2n + 2}}] iff [alpha + {sqrt{2n + 1}}] = [alpha + {sqrt{2n + 2}}] + 1 iff [alpha + {sqrt{2n + 1}}] = [alpha] + 1 iff [alpha + {sqrt{2n + 1}}] = 1$

From this we get that $alpha = 1 - {sqrt{2n + 1}} tag 2$



Now we prove the following: (this is where it gets really ambiguous)

There does exists $n_1, n_2 in mathbb{N}$, $n_1 ne frac {{p_1}^2} {2} - 1$, $n_2 = frac {{p_2}^2} {2} - 1$, $p_1$, $p_2 in mathbb{N} iff [sqrt{2n_1 + 1}] = [sqrt{2n_1 + 2}]$, $[sqrt{2n_2 + 1}] ne [sqrt{2n_2 + 2}]$ such that $ 1 - {sqrt{2n_1 + 2}} le 1 - {sqrt{2n_2 + 1}} lt 1 - {sqrt{2n_1 + 1}} tag 3 iff$ there is no $alpha$ satisfying both $(1)$ and $(2)$.
$implies {sqrt{2n_1 + 1}} lt {sqrt{2n_2 + 1}} le {sqrt{2n_1 + 2}} tag 4$
(this is where it gets really-really-really ambiguous)




If we substitute $n_1 = 3$ and $n_2 = 1$ in $(4)$ we get the following:
${sqrt{7}} lt {sqrt{3}} le {sqrt{8}} implies approx 0.64 lt 0.73 le 0.82$ wich is indeed true. So we found two exceptions for $n$ and we should be done with the proof.




This part is strongly wrong! I made a mistake. $(4)$ is wrong. In fact, I can't find $n_1$ and $n_2$ satisfying $(3)$. Now I believe that I am wrong and there does exist a desired $a$.



Note: I made a lot of edits. My proof changed since I posted the question. Please take a look at the edits.










share|cite|improve this question











$endgroup$












  • $begingroup$
    @TheSimpliFire appears the quoted part in yellow should read "every" $n$ rather than "any" $n,$ as here the "any" is ambiguous. Now that I think of it, "all" is pretty good, then it reads "for all"
    $endgroup$
    – Will Jagy
    Jan 4 at 20:17












  • $begingroup$
    @TheSimpliFire I am afraid that one of us didn't understand something properly.
    $endgroup$
    – Krisztián Kiss
    Jan 4 at 20:18










  • $begingroup$
    @KrisztiánKiss I recommend you change the quoted phrase from "for any" to "for all"
    $endgroup$
    – Will Jagy
    Jan 4 at 20:21










  • $begingroup$
    @WillJagy thank you.
    $endgroup$
    – Krisztián Kiss
    Jan 4 at 20:23














1












1








1





$begingroup$


The problem is the following:




Does $a in mathbb{R}$ exist such that $[a + sqrt{2n + 1}] = [a + sqrt{2n + 2}]$ for all $n in mathbb{N}$? ($[x]$ denotes the whole part of $x$).




Note: I will also use ${x}$ to denote the fractional part of $x$. Also that $[x] + {x} = x$.



My approach is the following:



There is no such that $a$.



Proof: For the sake of simplicity, let's assume $a = k + alpha$, $k in mathbb{Z}$ and $0 le alpha lt 1$. Because of $[x + k] = [x] + k$ for any $k in mathbb{Z}$, and substituting $a = k + alpha$, our equation $[k + alpha + sqrt{2n + 1}] = [k + alpha + sqrt{2n + 2}]$ becomes $[alpha + sqrt{2n + 1}] = [alpha + sqrt{2n + 2}]$. Now our task is to find $alpha in [0, 1)$ that satisfies the equation for any $n in mathbb{N}$.



First, let's find a formula for $alpha$ for given $n$. There are two cases:



1. $[sqrt{2n + 1}] = [sqrt{2n + 2}] = m$, $m in mathbb{N}$. This happens when $2n + 2$ isn't a perfect square $iff$ $2n + 2 ne p^2 iff n ne frac {p^2} {2} - 1$, $p in mathbb{N}$.



$[alpha + sqrt{2n + 1}] = [alpha + sqrt{2n + 2}] iff [alpha + [sqrt{2n + 1}] + {sqrt{2n + 1}}] = [alpha + [sqrt{2n + 2}] + {sqrt{2n + 2}}] iff [alpha + {sqrt{2n + 1}} + m] = [alpha + {sqrt{2n + 2}} + m] iff [alpha + {sqrt{2n + 1}}] = [alpha + {sqrt{2n + 2}}]$.

From this we get that $alpha in [0, 1 - {sqrt{2n + 2}}) tag {1'}$
Edit #1 (no longer actual beacuse of Edit #2): took out the maximum function from $(1')$ for obvious reason: the fractional part function is monotonically growing on its period, so if $[x] = [y]$, then ${x} lt {y}$ only if $x lt y$.
Edit #2: $(1')$ is only a partial solution for the 1. case, it is indeed a solution if both sides of the latest equation is equal to $0$. But they can both equal to $1$, so we have to consider that $alpha in [1 - {sqrt{2n + 1}}, 1)$. 1. case overall:
$alpha in [0, 1 - {sqrt{2n + 2}}) cup [1 - {sqrt{2n + 1}}, 1) tag 1$



2. $[sqrt{2n + 1}] ne [sqrt{2n + 2}] implies [sqrt{2n + 1}] = m, [sqrt{2n + 2}] = m + 1$, $m in mathbb{N}$. This happens when $2n + 2$ is a perfect square $iff$ $2n + 2 = p^2 iff n = frac {p^2} {2} - 1$, $p in mathbb{N}$. Now $[sqrt{2n + 2}] = sqrt{2n + 2} implies {sqrt{2n + 2}} = 0$.



$[alpha + sqrt{2n + 1}] = [alpha + sqrt{2n + 2}] iff [alpha + [sqrt{2n + 1}] + {sqrt{2n + 1}}] = [alpha + [sqrt{2n + 2}] + {sqrt{2n + 2}}] iff [alpha + {sqrt{2n + 1}}] = [alpha + {sqrt{2n + 2}}] + 1 iff [alpha + {sqrt{2n + 1}}] = [alpha] + 1 iff [alpha + {sqrt{2n + 1}}] = 1$

From this we get that $alpha = 1 - {sqrt{2n + 1}} tag 2$



Now we prove the following: (this is where it gets really ambiguous)

There does exists $n_1, n_2 in mathbb{N}$, $n_1 ne frac {{p_1}^2} {2} - 1$, $n_2 = frac {{p_2}^2} {2} - 1$, $p_1$, $p_2 in mathbb{N} iff [sqrt{2n_1 + 1}] = [sqrt{2n_1 + 2}]$, $[sqrt{2n_2 + 1}] ne [sqrt{2n_2 + 2}]$ such that $ 1 - {sqrt{2n_1 + 2}} le 1 - {sqrt{2n_2 + 1}} lt 1 - {sqrt{2n_1 + 1}} tag 3 iff$ there is no $alpha$ satisfying both $(1)$ and $(2)$.
$implies {sqrt{2n_1 + 1}} lt {sqrt{2n_2 + 1}} le {sqrt{2n_1 + 2}} tag 4$
(this is where it gets really-really-really ambiguous)




If we substitute $n_1 = 3$ and $n_2 = 1$ in $(4)$ we get the following:
${sqrt{7}} lt {sqrt{3}} le {sqrt{8}} implies approx 0.64 lt 0.73 le 0.82$ wich is indeed true. So we found two exceptions for $n$ and we should be done with the proof.




This part is strongly wrong! I made a mistake. $(4)$ is wrong. In fact, I can't find $n_1$ and $n_2$ satisfying $(3)$. Now I believe that I am wrong and there does exist a desired $a$.



Note: I made a lot of edits. My proof changed since I posted the question. Please take a look at the edits.










share|cite|improve this question











$endgroup$




The problem is the following:




Does $a in mathbb{R}$ exist such that $[a + sqrt{2n + 1}] = [a + sqrt{2n + 2}]$ for all $n in mathbb{N}$? ($[x]$ denotes the whole part of $x$).




Note: I will also use ${x}$ to denote the fractional part of $x$. Also that $[x] + {x} = x$.



My approach is the following:



There is no such that $a$.



Proof: For the sake of simplicity, let's assume $a = k + alpha$, $k in mathbb{Z}$ and $0 le alpha lt 1$. Because of $[x + k] = [x] + k$ for any $k in mathbb{Z}$, and substituting $a = k + alpha$, our equation $[k + alpha + sqrt{2n + 1}] = [k + alpha + sqrt{2n + 2}]$ becomes $[alpha + sqrt{2n + 1}] = [alpha + sqrt{2n + 2}]$. Now our task is to find $alpha in [0, 1)$ that satisfies the equation for any $n in mathbb{N}$.



First, let's find a formula for $alpha$ for given $n$. There are two cases:



1. $[sqrt{2n + 1}] = [sqrt{2n + 2}] = m$, $m in mathbb{N}$. This happens when $2n + 2$ isn't a perfect square $iff$ $2n + 2 ne p^2 iff n ne frac {p^2} {2} - 1$, $p in mathbb{N}$.



$[alpha + sqrt{2n + 1}] = [alpha + sqrt{2n + 2}] iff [alpha + [sqrt{2n + 1}] + {sqrt{2n + 1}}] = [alpha + [sqrt{2n + 2}] + {sqrt{2n + 2}}] iff [alpha + {sqrt{2n + 1}} + m] = [alpha + {sqrt{2n + 2}} + m] iff [alpha + {sqrt{2n + 1}}] = [alpha + {sqrt{2n + 2}}]$.

From this we get that $alpha in [0, 1 - {sqrt{2n + 2}}) tag {1'}$
Edit #1 (no longer actual beacuse of Edit #2): took out the maximum function from $(1')$ for obvious reason: the fractional part function is monotonically growing on its period, so if $[x] = [y]$, then ${x} lt {y}$ only if $x lt y$.
Edit #2: $(1')$ is only a partial solution for the 1. case, it is indeed a solution if both sides of the latest equation is equal to $0$. But they can both equal to $1$, so we have to consider that $alpha in [1 - {sqrt{2n + 1}}, 1)$. 1. case overall:
$alpha in [0, 1 - {sqrt{2n + 2}}) cup [1 - {sqrt{2n + 1}}, 1) tag 1$



2. $[sqrt{2n + 1}] ne [sqrt{2n + 2}] implies [sqrt{2n + 1}] = m, [sqrt{2n + 2}] = m + 1$, $m in mathbb{N}$. This happens when $2n + 2$ is a perfect square $iff$ $2n + 2 = p^2 iff n = frac {p^2} {2} - 1$, $p in mathbb{N}$. Now $[sqrt{2n + 2}] = sqrt{2n + 2} implies {sqrt{2n + 2}} = 0$.



$[alpha + sqrt{2n + 1}] = [alpha + sqrt{2n + 2}] iff [alpha + [sqrt{2n + 1}] + {sqrt{2n + 1}}] = [alpha + [sqrt{2n + 2}] + {sqrt{2n + 2}}] iff [alpha + {sqrt{2n + 1}}] = [alpha + {sqrt{2n + 2}}] + 1 iff [alpha + {sqrt{2n + 1}}] = [alpha] + 1 iff [alpha + {sqrt{2n + 1}}] = 1$

From this we get that $alpha = 1 - {sqrt{2n + 1}} tag 2$



Now we prove the following: (this is where it gets really ambiguous)

There does exists $n_1, n_2 in mathbb{N}$, $n_1 ne frac {{p_1}^2} {2} - 1$, $n_2 = frac {{p_2}^2} {2} - 1$, $p_1$, $p_2 in mathbb{N} iff [sqrt{2n_1 + 1}] = [sqrt{2n_1 + 2}]$, $[sqrt{2n_2 + 1}] ne [sqrt{2n_2 + 2}]$ such that $ 1 - {sqrt{2n_1 + 2}} le 1 - {sqrt{2n_2 + 1}} lt 1 - {sqrt{2n_1 + 1}} tag 3 iff$ there is no $alpha$ satisfying both $(1)$ and $(2)$.
$implies {sqrt{2n_1 + 1}} lt {sqrt{2n_2 + 1}} le {sqrt{2n_1 + 2}} tag 4$
(this is where it gets really-really-really ambiguous)




If we substitute $n_1 = 3$ and $n_2 = 1$ in $(4)$ we get the following:
${sqrt{7}} lt {sqrt{3}} le {sqrt{8}} implies approx 0.64 lt 0.73 le 0.82$ wich is indeed true. So we found two exceptions for $n$ and we should be done with the proof.




This part is strongly wrong! I made a mistake. $(4)$ is wrong. In fact, I can't find $n_1$ and $n_2$ satisfying $(3)$. Now I believe that I am wrong and there does exist a desired $a$.



Note: I made a lot of edits. My proof changed since I posted the question. Please take a look at the edits.







proof-verification proof-writing floor-function






share|cite|improve this question















share|cite|improve this question













share|cite|improve this question




share|cite|improve this question








edited Jan 5 at 11:08







Krisztián Kiss

















asked Jan 4 at 20:06









Krisztián KissKrisztián Kiss

484




484












  • $begingroup$
    @TheSimpliFire appears the quoted part in yellow should read "every" $n$ rather than "any" $n,$ as here the "any" is ambiguous. Now that I think of it, "all" is pretty good, then it reads "for all"
    $endgroup$
    – Will Jagy
    Jan 4 at 20:17












  • $begingroup$
    @TheSimpliFire I am afraid that one of us didn't understand something properly.
    $endgroup$
    – Krisztián Kiss
    Jan 4 at 20:18










  • $begingroup$
    @KrisztiánKiss I recommend you change the quoted phrase from "for any" to "for all"
    $endgroup$
    – Will Jagy
    Jan 4 at 20:21










  • $begingroup$
    @WillJagy thank you.
    $endgroup$
    – Krisztián Kiss
    Jan 4 at 20:23


















  • $begingroup$
    @TheSimpliFire appears the quoted part in yellow should read "every" $n$ rather than "any" $n,$ as here the "any" is ambiguous. Now that I think of it, "all" is pretty good, then it reads "for all"
    $endgroup$
    – Will Jagy
    Jan 4 at 20:17












  • $begingroup$
    @TheSimpliFire I am afraid that one of us didn't understand something properly.
    $endgroup$
    – Krisztián Kiss
    Jan 4 at 20:18










  • $begingroup$
    @KrisztiánKiss I recommend you change the quoted phrase from "for any" to "for all"
    $endgroup$
    – Will Jagy
    Jan 4 at 20:21










  • $begingroup$
    @WillJagy thank you.
    $endgroup$
    – Krisztián Kiss
    Jan 4 at 20:23
















$begingroup$
@TheSimpliFire appears the quoted part in yellow should read "every" $n$ rather than "any" $n,$ as here the "any" is ambiguous. Now that I think of it, "all" is pretty good, then it reads "for all"
$endgroup$
– Will Jagy
Jan 4 at 20:17






$begingroup$
@TheSimpliFire appears the quoted part in yellow should read "every" $n$ rather than "any" $n,$ as here the "any" is ambiguous. Now that I think of it, "all" is pretty good, then it reads "for all"
$endgroup$
– Will Jagy
Jan 4 at 20:17














$begingroup$
@TheSimpliFire I am afraid that one of us didn't understand something properly.
$endgroup$
– Krisztián Kiss
Jan 4 at 20:18




$begingroup$
@TheSimpliFire I am afraid that one of us didn't understand something properly.
$endgroup$
– Krisztián Kiss
Jan 4 at 20:18












$begingroup$
@KrisztiánKiss I recommend you change the quoted phrase from "for any" to "for all"
$endgroup$
– Will Jagy
Jan 4 at 20:21




$begingroup$
@KrisztiánKiss I recommend you change the quoted phrase from "for any" to "for all"
$endgroup$
– Will Jagy
Jan 4 at 20:21












$begingroup$
@WillJagy thank you.
$endgroup$
– Krisztián Kiss
Jan 4 at 20:23




$begingroup$
@WillJagy thank you.
$endgroup$
– Krisztián Kiss
Jan 4 at 20:23










2 Answers
2






active

oldest

votes


















0












$begingroup$

Your idea of looking at $alpha={a}$ is a good one.



If $alpha lt 2-sqrt 3approx 0.268$ we choose $n=1$ and note that $sqrt {2n+1}+alpha=sqrt 3+alpha$ has floor $1$ while $sqrt {2n+2}=sqrt 4=2$



If $alpha ge 2-sqrt 3$ we want to choose $n$ so that $$lflooralpha+sqrt {2n+2}rfloor = k gt lfloor alpha +sqrt {2n+1} rfloor\
lfloor alpha^2+2alphasqrt{2n+2}+2n+2 rfloor=k^2gt lfloor alpha^2+2alphasqrt{2n+1}+2n+1rfloor \
lfloor alpha^2+2alphasqrt{2n+2}+1 rfloor=k^2gt lfloor alpha^2+2alphasqrt{2n+1}rfloor $$

We note that when $2n$ is a square $sqrt {2n+1}approx sqrt{2n}(1+frac 1{2n})$ (rounding up the Taylor series) so if we choose $sqrt{2n} gt frac 1alpha$ and $2n$ a square it will fail.






share|cite|improve this answer









$endgroup$





















    0












    $begingroup$

    Beautiful proof that there is a solution:




    You can choose $a=frac{1}{2}$. Then the equality holds for all $n in mathbb{N}$. We can prove it by contradiction. Assume there exists $n$ such that we don’t have equality. Then we can find $q in mathbb{N}$ such that
    $displaystyle frac{1}{2} + sqrt{2n+1} < q leq frac{1}{2} + sqrt{2n+2}$
    This is equivalent to
    $displaystyle 8n+4 < (2q-1)^2 leq 8n+8$
    In particular this implies that one of the four numbers $8n+5, 8n+6, 8n+7, 8n+8$ must be equal to $(2q-1)^2$.
    What is the remainder when we divide $(2q-1)^2$ by $8$? Well, we have
    $displaystyle (2q-1)^2 = 4q^2-4q+1 = 4q(q-1)+1.$
    Since one of the numbers $q-1$ and $q$ is even, the first summand is divisible by $8$ and hence $(2q-1)^2mod 8 =1$. But the other four numbers obviously leave remainders $5,6,7,$ and $8,$ respectively.
    So there we have our contradiction, meaning that no such $q$ can exist.




    Proof by Christian Schmidt on Quora. Link to my question there.






    share|cite|improve this answer









    $endgroup$













      Your Answer





      StackExchange.ifUsing("editor", function () {
      return StackExchange.using("mathjaxEditing", function () {
      StackExchange.MarkdownEditor.creationCallbacks.add(function (editor, postfix) {
      StackExchange.mathjaxEditing.prepareWmdForMathJax(editor, postfix, [["$", "$"], ["\\(","\\)"]]);
      });
      });
      }, "mathjax-editing");

      StackExchange.ready(function() {
      var channelOptions = {
      tags: "".split(" "),
      id: "69"
      };
      initTagRenderer("".split(" "), "".split(" "), channelOptions);

      StackExchange.using("externalEditor", function() {
      // Have to fire editor after snippets, if snippets enabled
      if (StackExchange.settings.snippets.snippetsEnabled) {
      StackExchange.using("snippets", function() {
      createEditor();
      });
      }
      else {
      createEditor();
      }
      });

      function createEditor() {
      StackExchange.prepareEditor({
      heartbeatType: 'answer',
      autoActivateHeartbeat: false,
      convertImagesToLinks: true,
      noModals: true,
      showLowRepImageUploadWarning: true,
      reputationToPostImages: 10,
      bindNavPrevention: true,
      postfix: "",
      imageUploader: {
      brandingHtml: "Powered by u003ca class="icon-imgur-white" href="https://imgur.com/"u003eu003c/au003e",
      contentPolicyHtml: "User contributions licensed under u003ca href="https://creativecommons.org/licenses/by-sa/3.0/"u003ecc by-sa 3.0 with attribution requiredu003c/au003e u003ca href="https://stackoverflow.com/legal/content-policy"u003e(content policy)u003c/au003e",
      allowUrls: true
      },
      noCode: true, onDemand: true,
      discardSelector: ".discard-answer"
      ,immediatelyShowMarkdownHelp:true
      });


      }
      });














      draft saved

      draft discarded


















      StackExchange.ready(
      function () {
      StackExchange.openid.initPostLogin('.new-post-login', 'https%3a%2f%2fmath.stackexchange.com%2fquestions%2f3062047%2fis-my-approach-correct-to-this-equation%23new-answer', 'question_page');
      }
      );

      Post as a guest















      Required, but never shown

























      2 Answers
      2






      active

      oldest

      votes








      2 Answers
      2






      active

      oldest

      votes









      active

      oldest

      votes






      active

      oldest

      votes









      0












      $begingroup$

      Your idea of looking at $alpha={a}$ is a good one.



      If $alpha lt 2-sqrt 3approx 0.268$ we choose $n=1$ and note that $sqrt {2n+1}+alpha=sqrt 3+alpha$ has floor $1$ while $sqrt {2n+2}=sqrt 4=2$



      If $alpha ge 2-sqrt 3$ we want to choose $n$ so that $$lflooralpha+sqrt {2n+2}rfloor = k gt lfloor alpha +sqrt {2n+1} rfloor\
      lfloor alpha^2+2alphasqrt{2n+2}+2n+2 rfloor=k^2gt lfloor alpha^2+2alphasqrt{2n+1}+2n+1rfloor \
      lfloor alpha^2+2alphasqrt{2n+2}+1 rfloor=k^2gt lfloor alpha^2+2alphasqrt{2n+1}rfloor $$

      We note that when $2n$ is a square $sqrt {2n+1}approx sqrt{2n}(1+frac 1{2n})$ (rounding up the Taylor series) so if we choose $sqrt{2n} gt frac 1alpha$ and $2n$ a square it will fail.






      share|cite|improve this answer









      $endgroup$


















        0












        $begingroup$

        Your idea of looking at $alpha={a}$ is a good one.



        If $alpha lt 2-sqrt 3approx 0.268$ we choose $n=1$ and note that $sqrt {2n+1}+alpha=sqrt 3+alpha$ has floor $1$ while $sqrt {2n+2}=sqrt 4=2$



        If $alpha ge 2-sqrt 3$ we want to choose $n$ so that $$lflooralpha+sqrt {2n+2}rfloor = k gt lfloor alpha +sqrt {2n+1} rfloor\
        lfloor alpha^2+2alphasqrt{2n+2}+2n+2 rfloor=k^2gt lfloor alpha^2+2alphasqrt{2n+1}+2n+1rfloor \
        lfloor alpha^2+2alphasqrt{2n+2}+1 rfloor=k^2gt lfloor alpha^2+2alphasqrt{2n+1}rfloor $$

        We note that when $2n$ is a square $sqrt {2n+1}approx sqrt{2n}(1+frac 1{2n})$ (rounding up the Taylor series) so if we choose $sqrt{2n} gt frac 1alpha$ and $2n$ a square it will fail.






        share|cite|improve this answer









        $endgroup$
















          0












          0








          0





          $begingroup$

          Your idea of looking at $alpha={a}$ is a good one.



          If $alpha lt 2-sqrt 3approx 0.268$ we choose $n=1$ and note that $sqrt {2n+1}+alpha=sqrt 3+alpha$ has floor $1$ while $sqrt {2n+2}=sqrt 4=2$



          If $alpha ge 2-sqrt 3$ we want to choose $n$ so that $$lflooralpha+sqrt {2n+2}rfloor = k gt lfloor alpha +sqrt {2n+1} rfloor\
          lfloor alpha^2+2alphasqrt{2n+2}+2n+2 rfloor=k^2gt lfloor alpha^2+2alphasqrt{2n+1}+2n+1rfloor \
          lfloor alpha^2+2alphasqrt{2n+2}+1 rfloor=k^2gt lfloor alpha^2+2alphasqrt{2n+1}rfloor $$

          We note that when $2n$ is a square $sqrt {2n+1}approx sqrt{2n}(1+frac 1{2n})$ (rounding up the Taylor series) so if we choose $sqrt{2n} gt frac 1alpha$ and $2n$ a square it will fail.






          share|cite|improve this answer









          $endgroup$



          Your idea of looking at $alpha={a}$ is a good one.



          If $alpha lt 2-sqrt 3approx 0.268$ we choose $n=1$ and note that $sqrt {2n+1}+alpha=sqrt 3+alpha$ has floor $1$ while $sqrt {2n+2}=sqrt 4=2$



          If $alpha ge 2-sqrt 3$ we want to choose $n$ so that $$lflooralpha+sqrt {2n+2}rfloor = k gt lfloor alpha +sqrt {2n+1} rfloor\
          lfloor alpha^2+2alphasqrt{2n+2}+2n+2 rfloor=k^2gt lfloor alpha^2+2alphasqrt{2n+1}+2n+1rfloor \
          lfloor alpha^2+2alphasqrt{2n+2}+1 rfloor=k^2gt lfloor alpha^2+2alphasqrt{2n+1}rfloor $$

          We note that when $2n$ is a square $sqrt {2n+1}approx sqrt{2n}(1+frac 1{2n})$ (rounding up the Taylor series) so if we choose $sqrt{2n} gt frac 1alpha$ and $2n$ a square it will fail.







          share|cite|improve this answer












          share|cite|improve this answer



          share|cite|improve this answer










          answered Jan 4 at 22:12









          Ross MillikanRoss Millikan

          298k24200373




          298k24200373























              0












              $begingroup$

              Beautiful proof that there is a solution:




              You can choose $a=frac{1}{2}$. Then the equality holds for all $n in mathbb{N}$. We can prove it by contradiction. Assume there exists $n$ such that we don’t have equality. Then we can find $q in mathbb{N}$ such that
              $displaystyle frac{1}{2} + sqrt{2n+1} < q leq frac{1}{2} + sqrt{2n+2}$
              This is equivalent to
              $displaystyle 8n+4 < (2q-1)^2 leq 8n+8$
              In particular this implies that one of the four numbers $8n+5, 8n+6, 8n+7, 8n+8$ must be equal to $(2q-1)^2$.
              What is the remainder when we divide $(2q-1)^2$ by $8$? Well, we have
              $displaystyle (2q-1)^2 = 4q^2-4q+1 = 4q(q-1)+1.$
              Since one of the numbers $q-1$ and $q$ is even, the first summand is divisible by $8$ and hence $(2q-1)^2mod 8 =1$. But the other four numbers obviously leave remainders $5,6,7,$ and $8,$ respectively.
              So there we have our contradiction, meaning that no such $q$ can exist.




              Proof by Christian Schmidt on Quora. Link to my question there.






              share|cite|improve this answer









              $endgroup$


















                0












                $begingroup$

                Beautiful proof that there is a solution:




                You can choose $a=frac{1}{2}$. Then the equality holds for all $n in mathbb{N}$. We can prove it by contradiction. Assume there exists $n$ such that we don’t have equality. Then we can find $q in mathbb{N}$ such that
                $displaystyle frac{1}{2} + sqrt{2n+1} < q leq frac{1}{2} + sqrt{2n+2}$
                This is equivalent to
                $displaystyle 8n+4 < (2q-1)^2 leq 8n+8$
                In particular this implies that one of the four numbers $8n+5, 8n+6, 8n+7, 8n+8$ must be equal to $(2q-1)^2$.
                What is the remainder when we divide $(2q-1)^2$ by $8$? Well, we have
                $displaystyle (2q-1)^2 = 4q^2-4q+1 = 4q(q-1)+1.$
                Since one of the numbers $q-1$ and $q$ is even, the first summand is divisible by $8$ and hence $(2q-1)^2mod 8 =1$. But the other four numbers obviously leave remainders $5,6,7,$ and $8,$ respectively.
                So there we have our contradiction, meaning that no such $q$ can exist.




                Proof by Christian Schmidt on Quora. Link to my question there.






                share|cite|improve this answer









                $endgroup$
















                  0












                  0








                  0





                  $begingroup$

                  Beautiful proof that there is a solution:




                  You can choose $a=frac{1}{2}$. Then the equality holds for all $n in mathbb{N}$. We can prove it by contradiction. Assume there exists $n$ such that we don’t have equality. Then we can find $q in mathbb{N}$ such that
                  $displaystyle frac{1}{2} + sqrt{2n+1} < q leq frac{1}{2} + sqrt{2n+2}$
                  This is equivalent to
                  $displaystyle 8n+4 < (2q-1)^2 leq 8n+8$
                  In particular this implies that one of the four numbers $8n+5, 8n+6, 8n+7, 8n+8$ must be equal to $(2q-1)^2$.
                  What is the remainder when we divide $(2q-1)^2$ by $8$? Well, we have
                  $displaystyle (2q-1)^2 = 4q^2-4q+1 = 4q(q-1)+1.$
                  Since one of the numbers $q-1$ and $q$ is even, the first summand is divisible by $8$ and hence $(2q-1)^2mod 8 =1$. But the other four numbers obviously leave remainders $5,6,7,$ and $8,$ respectively.
                  So there we have our contradiction, meaning that no such $q$ can exist.




                  Proof by Christian Schmidt on Quora. Link to my question there.






                  share|cite|improve this answer









                  $endgroup$



                  Beautiful proof that there is a solution:




                  You can choose $a=frac{1}{2}$. Then the equality holds for all $n in mathbb{N}$. We can prove it by contradiction. Assume there exists $n$ such that we don’t have equality. Then we can find $q in mathbb{N}$ such that
                  $displaystyle frac{1}{2} + sqrt{2n+1} < q leq frac{1}{2} + sqrt{2n+2}$
                  This is equivalent to
                  $displaystyle 8n+4 < (2q-1)^2 leq 8n+8$
                  In particular this implies that one of the four numbers $8n+5, 8n+6, 8n+7, 8n+8$ must be equal to $(2q-1)^2$.
                  What is the remainder when we divide $(2q-1)^2$ by $8$? Well, we have
                  $displaystyle (2q-1)^2 = 4q^2-4q+1 = 4q(q-1)+1.$
                  Since one of the numbers $q-1$ and $q$ is even, the first summand is divisible by $8$ and hence $(2q-1)^2mod 8 =1$. But the other four numbers obviously leave remainders $5,6,7,$ and $8,$ respectively.
                  So there we have our contradiction, meaning that no such $q$ can exist.




                  Proof by Christian Schmidt on Quora. Link to my question there.







                  share|cite|improve this answer












                  share|cite|improve this answer



                  share|cite|improve this answer










                  answered Jan 5 at 11:15









                  Krisztián KissKrisztián Kiss

                  484




                  484






























                      draft saved

                      draft discarded




















































                      Thanks for contributing an answer to Mathematics Stack Exchange!


                      • Please be sure to answer the question. Provide details and share your research!

                      But avoid



                      • Asking for help, clarification, or responding to other answers.

                      • Making statements based on opinion; back them up with references or personal experience.


                      Use MathJax to format equations. MathJax reference.


                      To learn more, see our tips on writing great answers.




                      draft saved


                      draft discarded














                      StackExchange.ready(
                      function () {
                      StackExchange.openid.initPostLogin('.new-post-login', 'https%3a%2f%2fmath.stackexchange.com%2fquestions%2f3062047%2fis-my-approach-correct-to-this-equation%23new-answer', 'question_page');
                      }
                      );

                      Post as a guest















                      Required, but never shown





















































                      Required, but never shown














                      Required, but never shown












                      Required, but never shown







                      Required, but never shown

































                      Required, but never shown














                      Required, but never shown












                      Required, but never shown







                      Required, but never shown







                      Popular posts from this blog

                      Quarter-circle Tiles

                      build a pushdown automaton that recognizes the reverse language of a given pushdown automaton?

                      Mont Emei